A box has a height of 5 cm a Length
os 6 cm and a width of 2 cm
What is its volumes​

Answers

Answer 1
The box has a volume of 5x6x2=60cm^3

Related Questions

how to write "The product of a number and -2 increased by 10"? ​

Answers

(x•-2)+10 I’m pretty sure
Possible ways

2(x) + 10

x • 2 + 10

x(2) + 10

What is the slope of the line that contains these points?
x
-4
1
6
11
y
-3
-2
-1
0

Answers

Answer:

Step-by-step explanation:

The slope of the line y = (x/4) - 23/4 is 1/4.

What is a slope?

In mathematics, a line's slope, also known as its gradient, is a numerical representation of the line's steepness and direction.

Given:

The points,

x → y

-4 → -3

1   →  -2

6 →   -1

11  →   0

From the table, we interpreted 2 points (-4, -3) and (11, 0)

If a line passes through two points (x₁ ,y₁) and (x₂, y₂),

then the equation of line is

y - y₁ = (y₂- y₁) / (x₂ - x₁) x (x - x₁)

To find the slope;

m =  (y₂- y₁) / (x₂ - x₁)

m = (0+3) / (11+4)

m = 1/5

So, the equation of line is

y - y₁ = (y₂- y₁) / (x₂ - x₁)  (x - x₁)

y - y₁ =m (x - x₁)

y +3 = (1/4) (x -11)

y = (x / 4) - 11/4 -3

y = (x/4) - 23/4

Therefore, the slope of the line is m = 1/5.

To learn more about the slope;

brainly.com/question/3605446

#SPJ2

Ms. Lynette earns $19.50 an hour when she works overtime. She worked overtime twice this week. One day she worked 3 hours of overtime. Her total overtime pay for the week is $146.25.

Write an equation to find the number of overtime hours she worked on the second day.

Answers

(19.50*3)+(19.50*x)=146.25

Answer:

4 1/2 hours

Step-by-step explanation:

Equation(s):

--

(146.25 - (19.50 x 3)) ÷ 19.50 =

4.5 or 4 1/2 hours

--

#LFM #Brainly

Which statement is true?
The following are the amounts, in dollars, that 15 items
sold for at a silent auction as part of a school fundraiser.
15, 24, 89, 76, 45, 32, 55, 140,
72, 21, 52, 68, 81, 33, 40
O Only 15 is an outlier.
o Only 140 is an outlier.
O Both 15 and 140 are outliers.
There are no outliers.

Answers

The statement which is true among the answer choices is; Only 140 is an outlier.

Which data value among the given values is an outlier?

An outlier by definition is a data point that differs significantly from other observations which may be due to an error and hence, may cause difficulty in statistical evaluation.

On this note, it follows that only 140 is an outlier among the answer choices.

Read more on outliers;

https://brainly.com/question/3631910

#SPJ1

It's D. There are no outliers

80% of n is 32 what is the value of n​

Answers

Answer:

n=40

Step-by-step explanation:

divide 32 by 80

32/80=0.4

0.4x100=40

n=40



A bag contains 2 5/8 pounds of candy. If the candy is equally distributed to point
seven children, how many pounds will each child get?"
3/7
3/8
3/9
3/10

Answers

The correct answer is 3/8

26n- 7m+4 (10n-6m) be rewritten

Answers

Answer:

260n^2-226nm+42m^2+40n-24m

Step-by-step explanation:

26n-7m+4(10n-6m) use box method

Which function is equivalent to g of x = x squared + 6x + 8?
A.g(x)=(x+2)(x+4)
B. g(x)=(x-2)(x--4)
C. g(x)=(x+2)(x-4)
D.g(x)=(x-2)(x+4)

Answers

I think A, there are no negatives in this equation

An rising star starts out with a Twitter following 1200 and her follows increase

Answers

Answer:

How we solve the question and is that the complete question?

Step-by-step explanation:

can someone plz help

Answers

Answer:

-9.56

Step-by-step explanation:

plz help i really need it

n/5 - 3 = -6
A) -20
B) -2
C) 15
D) -15

Answers

The answer is D)!

Hope this helps :)

Answer:

D

Step-by-step explanation:

n/5 = -3

n = -3 x 5

n = -15

what is the surface area​

Answers

Answer:

The surface area of the rectangular prism is 748 ft2.

Step-by-step explanation:

formula for surface area: A=2(wl+hl+hw)

A=2(wl+hl+hw)=2(5x16+14x16+14x5)=748

Answer:

748 ft²

Step-by-step explanation:

For the surface area, you need to add up the area of all sides, six in this example, but it can be simplified into three by multiplying some of them by two as its presumed a rectangle

x1 = 5*14*2 = 140

x2 = 16*14*2 = 448

x3 =16*5*2 = 160

You find the sum of these; 140 + 160 + 448 = 300 + 448 = 748

find the difference between (14-3x) and (-9x+7)

Answers

Answer:

Step-by-step explanation:

The difference between 14 - 3x and - 9x + 7 is

= 14 - 3x - ( - 9x + 7)

= 14 - 3x + 9x - 7

= 6x + 7

hope it helps :)

Solve:
One week, seventy-two percent of the animals that came to a veterinary hospital were dogs. If there were 230 animals total, how many were dogs?

Answers

Answer:

166 dogs were at the veterinary hospital

Step-by-step explanation:

Here, we want to know the number of dogs that were at the veterinary hospital

From the question, 72% of these animals are dogs

72% of a total of 230

Mathematically, the number of dogs are as follows;

dogs = 72% of 230

dogs = 72/100 * 230 = 165.6

since we cannot have a fractional dog, we round off to the nearest whole number which is 166 dogs

The points D(9, 8)and E(3, 13) are on a coordinate plane. What is theapproximate distance between points D and E?

Answers

Answer:7.8102 is the distance between those two points

Step-by-step explanation:

If you are good at geometry pls help

Answers

bro copy the whole problem and paste it in brainly you’ll get the right answer works every time !!!!

Find the value of x in the angle pair.

Answers

Answer:

but I don't want you guys

Step-by-step explanation:

ok love to have a God I love you night with my friend who is a control needed for a while and he was very excited to see if he says that you were a great depression in his life when you had a good time and you were going from a big it would have a lot of fun and I just love that I feel pretty

Answer:

10

Step-by-step explanation:

See the angles on the black line. They are supplementary.

7x+90°+20°=180°

7x+110°=180°

7x=180°-110°

7x=70°

x=70°/7

x=10

A vidoe game costs 29.35 dollars at one store and 32.79 at another store genarte two equivelnts expressions and then find how much more 4 video games cost at the second store

Answers

Answer:

$13.76

Step-by-step explanation:

Cost at store A = $29.35

Cost at store B = $32.79

Difference in price at both stores :

Difference = store B - store A

Difference in price = $32.79 - $29.35 = $3.44

Extra cost to f purchasing 4 video games at store B:

Difference in price * number of video games

$3.44 * 4 = $13.76

Find a, b and c giving reasons for each answer.
2)
3)
1)
55
b
С
b
b
110°
с
a
a
С
120°
5)
6)
25 °
4)
115
a
10°
ab

Answers

Answer/Step-by-step explanation:

1.

✔️a = 180 - 110 (consecutive angles are supplementary)

a = 70°

✔️b = a (corresponding angles are congruent)

b = 70°

✔️c = 110° (alternate interior angles are congruent)

2.

✔️a = 180 - 120 (linear pair)

a = 60°

✔️b = a (corresponding angles are congruent)

b = 60°

✔️c = a (alternate interior angles are congruent)

c = 60°

3.

✔️a = 55° (corresponding angles are congruent)

✔️b = 55° (Vertical angles are congruent)

✔️c = 180 - 55 = 125° (linear pair)

4.

✔️a = 115° (corresponding angles are congruent)

✔️b = 180 - a (linear pair)

b = 180 - 115

b = 65°

✔️c = b (vertical angles are congruent)

c = 65°

5.

✔️a = 180 - 25 = 155° (supplementary angles)

✔️b = 25° (corresponding angles)

✔️c = 25° (vertical angles)

6.

✔️a = 180 - 10 = 170° (linear pair)

✔️b = a (corresponding angles are congruent)

b = 170°

✔️c = 10° (corresponding angles are congruent)

Of the questions on his math test, Shane answered 42 correctly and 18 incorrectly. What percent of the total questions did Shane answer correctly?​

Answers

Answer:

She answered 70% of the questions correctly.

I need help with all of them

Answers

Answer:

Question 3: (D) All real numbers. Question 4: (B) y > 3.

Which of the following options have the same value as 62\%62%62, percent of 454545

Answers

Answer:

Here, the given expression is,

62 % of 45

  ( 1 % =  )

  ( Dividing 62 by 100 )

Hence, Option A and C are correct

The only thing that is the one is %

3(x+5) - 3 = 2(3x + 1) - 3
how many solutions does this equation have?

Answers

Answer:

Step-by-step explanation:

1 solution: x=13/3

I WILL DO ANYTHING JUST PLEASE

Answers

Answer:

95 0r 91

Step-by-step explanation:

Answer:

93

Step-by-step explanation:

trust me

please help i dont know what to do

Answers

Answer:

-15 + -3 = -18

Rob owes his father $18.

Step-by-step explanation:

Rob has to pay back the same amount of money that he took. Since he took 15 dollars, he has to give back 15 dollars which makes the 15 negative. The same goes for the 3 in the equation.

To find out the total amount Rob has to pay back, we have to add -15 and -3, which is -18.

15 points
What is the equation of the graph below?

Answers

We need to see all the given answers

Kelly has a rectangular fish aquarium that
measures 18 inches, 8 inches wide and 12 inches
tall. How much water does the aquarium can
hold?

Answers

volume = 18x8x12
= 1728

WILL GIBE BRAINLEST

The figure is made up of two cones and a cylinder. Both cones and the cylinder have a 10 mm diameter. What is the exact volume of this figure? What is the volume of this figure? A.250(pi)mm³
B.4007(pi)mm³
C.6257(pi)mm³
D.2500(pi)mm³​

Answers

Answer:

Step-by-step explanation:

The answer choices don’t make sense. The volume of a cylinder of height 45mm and radius 5mm = π5²*45 = 1125π mm³.

Three of the answer choices are larger than 1125π.

:::::

volume of one cone = πr²h* ⅓

Volume of cylinder section = πr²h

Total volume = πr²h*⅔ + πr²h = πr²h*5/3 = π5²*15*5/3 = 625π

A transformation of AKLM results in AK'L'M'.
Which transformation maps the pre-image to the
image?

dilation

translation

reflection

rotation

Answers

Translation, Dilation

Answer:

dilation

Step-by-step explanation:

took the test

PLEASE HELPPPPPPPPPPP​

Answers

Answer:

I think it is 20

Step-by-step explanation:

15+15+24+26=80÷4=20

Other Questions
Mi hermanapregunta la hora a my a mi pap. *mete- lenosles Suppose you deposited $200 in a savings account 4 years ago. The simple interest rate is 2.1%. The interest that you earned in those 4 years is $16.80. Which of the following is/are true? Select all that apply. A. r = 2.1%B. t = 4C. p = 16.80D. l = 200 Help plz!!,!!!!!!,!!!!!,!,!,!, Which sum represents the partial fraction decomposition of 8x^ 4 + 3x^3- 115x^2- 39x+200/ 3x (x^2-10)^2 grandma must add the areas of two plots of land to determine the amount in of seedy plant in her garden. the area of plot A can be represented by 3x^2 + 7x - 5, and the plot B can be represented by 5x^2 - 4x + 11. Write a polynomial that represents the total area of both plots of land. Al2(SO4)3 + 6 K --- > 3 K2SO4 + 2 AlIn an experiment, 46.8 grams of aluminum sulfate is reacted to make how many grams of potassium sulfate? Companies recognize revenue when goods or services are transferred to customers for the amount the company expects to be entitled to receive in exchange for those goods or services. That core principle is implemented by (1) identifying a contract with a customer, (2) identifying the performance obligations in the contract, (3) determining the transaction price of the contract, (4) allocating that price to the performance obligations, and (5) recognizing revenue when (or as) each performance obligation is satisfied. PLEASE HELP ASAP!!!!! Please Help! I'll give brainliest! explain how to answer the word problem below. 3 feet = 1 yard John has 476 feet of yarn and he needs to know how many yards he has. How many yards of yarn does John have. (solve and explain) Why is it important for gametes to go through meiosis Colombia's plains and coastal lowlands are ______.ASAP I WILL MAKE BRAINLIESTwarmtemperatehotcold Danny purchased a large wheel of cheese at the farmer's market.What is the volume of the cheese, to the nearest cubic inch ? The legislative branch of the national government is responsible for which of the following?OA. vetoing lawsOB. creating treatiesOC. nullifying lawsOD. imposing taxes Help Please !!A baby bird can eat 14 feet of earthworm every day. At this rate, how many days would it take six baby birds to eat 840 feet of worms? Two rectangles are similar. Which proportioncould you solve to find the missing side length?ILL GIVE U BRAINLEST PLEASE HELP ME Q1. Peter bought 3slices of chicken pizza and 4 slices of mushroom pizza for a total of $12.50.Mary bought 3 slices of chicken pizza and 2 slices of mushroom pizza for a total cost of $8.50.What is the cost of one slice of chicken and one slice of mushroom?PLS HELP I need help!!!!!!!!!!!! What empire fell when it was attacked by outside invaders Choose the answer that tells the location of a subordinate clause in the sentence.Sitting on a fence, the cat watched the activities in the barn.A. preceding the independent clauseB. in the middle of the independent clauseC. following the independent clauseD. There is no subordinate clause.